Difference between revisions of "2020 AMC 10A Problems/Problem 22"
(→Solution (Casework)) |
(→Solution (Casework)) |
||
Line 28: | Line 28: | ||
Note that <math>n = 1</math> doesn't work; to prove this, we just have to substitute <math>1</math> for <math>n</math> in the expression. | Note that <math>n = 1</math> doesn't work; to prove this, we just have to substitute <math>1</math> for <math>n</math> in the expression. | ||
This gives us | This gives us | ||
− | < | + | <math>\left\lfloor \dfrac{998}{1} \right\rfloor+\left\lfloor \dfrac{999}{1} \right\rfloor+\left\lfloor \dfrac{1000}{1}\right \rfloor</math> = \frac{998}1 + \frac{999}1 + \frac{1000}1 = 2997 = 999 \cdot 3<math> which is divisible by 3. |
− | Therefore, the case <math>n = 1< | + | Therefore, the case </math>n = 1<math> does not work. |
Line 35: | Line 35: | ||
− | <b>Case 1:</b> <math>n< | + | <b>Case 1:</b> </math>n<math> divides </math>998<math> |
− | The first case does not work, as the three terms in the expression must be <math>(a, a, a)< | + | The first case does not work, as the three terms in the expression must be </math>(a, a, a)<math>, as mentioned above, so the sum becomes </math>3a<math>, which is divisible by </math>3<math>. |
− | <b>Case 2:</b> <math>n< | + | <b>Case 2:</b> </math>n<math> divides </math>999<math> |
− | Because <math>n< | + | Because </math>n<math> divides </math>999<math>, the number of possibilities for </math>n<math> is the same as the number of factors of </math>999<math>, excluding </math>1<math>. |
− | <math>999< | + | </math>999<math> = </math>3^3 \cdot 37^1<math> |
− | So, the total number of factors of <math>999< | + | So, the total number of factors of </math>999<math> is </math>4 \cdot 2 = 8<math>. |
− | However, we have to subtract <math>1< | + | However, we have to subtract </math>1<math>, because the case </math>n = 1<math> doesn't work, as mentioned previously. |
− | <math>8 - 1 = 7< | + | </math>8 - 1 = 7<math> |
We now do the same for the third and last case. | We now do the same for the third and last case. | ||
− | <b>Case 3:</b> <math>n< | + | <b>Case 3:</b> </math>n<math> divides </math>1000<math> |
− | Because <math>n< | + | Because </math>n<math> divides </math>1000<math>, the number of possibilities for </math>n<math> is the same as the number of factors of </math>1000<math>, excluding </math>1<math>. |
− | <math>1000< | + | </math>1000<math> = </math>5^3 \cdot 2^3<math> |
− | So, the total number of factors of <math>1000< | + | So, the total number of factors of </math>1000<math> is </math>4 \cdot 4 = 16<math>. |
− | Again, we have to subtract <math>1< | + | Again, we have to subtract </math>1<math>, for the reason stated in Case 2. |
− | <math>16 - 1 = 15< | + | </math>16 - 1 = 15<math> |
Line 65: | Line 65: | ||
Now that we have counted all of the cases, we add them. | Now that we have counted all of the cases, we add them. | ||
− | <math>7 + 15 = 22< | + | </math>7 + 15 = 22<math>, so the answer is </math>\boxed{\textbf{(A)}22}$. |
Revision as of 21:24, 1 February 2020
Problem
For how many positive integers isnot divisible by ? (Recall that is the greatest integer less than or equal to .)
Solution (Casework)
Expression:
Solution:
Let
Notice that for every integer ,
if is an integer, then the three terms in the expression above must be ,
if is an integer, then the three terms in the expression above must be , and
if is an integer, then the three terms in the expression above must be .
This is due to the fact that , , and share no common factors collectively (other than 1).
Note that doesn't work; to prove this, we just have to substitute for in the expression.
This gives us
= \frac{998}1 + \frac{999}1 + \frac{1000}1 = 2997 = 999 \cdot 3n = 1$does not work.
Now, we test the three cases mentioned above.
<b>Case 1:</b>$ (Error compiling LaTeX. Unknown error_msg)n998(a, a, a)3a3$.
<b>Case 2:</b>$ (Error compiling LaTeX. Unknown error_msg)n999n999n99919993^3 \cdot 37^19994 \cdot 2 = 8$.
However, we have to subtract$ (Error compiling LaTeX. Unknown error_msg)1n = 18 - 1 = 7$We now do the same for the third and last case.
<b>Case 3:</b>$ (Error compiling LaTeX. Unknown error_msg)n1000n1000n1000110005^3 \cdot 2^310004 \cdot 4 = 16$.
Again, we have to subtract$ (Error compiling LaTeX. Unknown error_msg)116 - 1 = 157 + 15 = 22\boxed{\textbf{(A)}22}$.
~dragonchomper
Video Solution
~IceMatrix
See Also
2020 AMC 10A (Problems • Answer Key • Resources) | ||
Preceded by Problem 21 |
Followed by Problem 23 | |
1 • 2 • 3 • 4 • 5 • 6 • 7 • 8 • 9 • 10 • 11 • 12 • 13 • 14 • 15 • 16 • 17 • 18 • 19 • 20 • 21 • 22 • 23 • 24 • 25 | ||
All AMC 10 Problems and Solutions |
The problems on this page are copyrighted by the Mathematical Association of America's American Mathematics Competitions.